How Do You Calculate the Inverse Discrete Fourier Transform Matrix F(hat)?

Click For Summary
To calculate the inverse discrete Fourier transform matrix F(hat) for the given matrix F, first confirm that F is defined correctly as a 4x4 matrix with entries based on 5 raised to the power of the product of indices. The relationship F(hat)F = I indicates that F(hat) is the scaled inverse of F, specifically F(hat) = (1/4)F^-1. The i-th row of F(hat) can be constructed using the primitive root of unity, ω, leading to the form (1, ω^(-i), ω^(-2i), ω^(-3i)). This approach will help in calculating F(hat) accurately.
jmomo
Messages
8
Reaction score
0

Homework Statement


Let F be the 4x4 matrix whose (i, j)th entry is 5ij in F_13 for i, j = 0,1,2, 3.
Compute F(hat) and verify that F(hat)F = I


Homework Equations


The matrix F(hat) is called the inverse discrete Fourier transform of F.


The Attempt at a Solution


I found that e = 4, so (F)F(hat) = 4 I, so F(1/4 F(hat)) = I
I calculated that matrix F=
1 1 1 1
1 5 12 8
1 12 8 1
1 8 1 5

My Question: How do I calculate matrix F(hat)? I understand it is the inverse of F, but I am unsure of how to calculate it.
 
Physics news on Phys.org
jmomo said:

Homework Statement


Let F be the 4x4 matrix whose (i, j)th entry is 5ij in F_13 for i, j = 0,1,2, 3.
Does that mean anything to you? Because it doesn't to me.

Compute F(hat) and verify that F(hat)F = I


Homework Equations


The matrix F(hat) is called the inverse discrete Fourier transform of F.


The Attempt at a Solution


I found that e = 4, so (F)F(hat) = 4 I, so F(1/4 F(hat)) = I
I calculated that matrix F=
1 1 1 1
1 5 12 8
1 12 8 1
1 8 1 5

My Question: How do I calculate matrix F(hat)? I understand it is the inverse of F, but I am unsure of how to calculate it.
 
vela said:
Does that mean anything to you? Because it doesn't to me.

This question should have been a continuation of this thread:

https://www.physicsforums.com/showthread.php?t=751455

There the OP said he meant ##5^{i\cdot j}## instead of ##5ij##. Dunno why he didn't correct it for this post.
 
Might as well have continued this in your prior post. Now that you have ##F##, the ##i^{th}## row of ##\hat F## has the form:

$$(1, \omega^{-i}, \omega^{-2i}, ..., \omega^{-(e-1)i})$$

Where ##\omega## is the e'th primitive root of unity. I'm sure you can continue.
 
Question: A clock's minute hand has length 4 and its hour hand has length 3. What is the distance between the tips at the moment when it is increasing most rapidly?(Putnam Exam Question) Answer: Making assumption that both the hands moves at constant angular velocities, the answer is ## \sqrt{7} .## But don't you think this assumption is somewhat doubtful and wrong?

Similar threads

  • · Replies 2 ·
Replies
2
Views
2K
Replies
1
Views
2K
  • · Replies 6 ·
Replies
6
Views
2K
  • · Replies 5 ·
Replies
5
Views
2K
  • · Replies 4 ·
Replies
4
Views
3K
  • · Replies 4 ·
Replies
4
Views
8K
  • · Replies 1 ·
Replies
1
Views
1K
  • · Replies 5 ·
Replies
5
Views
3K
Replies
1
Views
2K
  • · Replies 17 ·
Replies
17
Views
3K